Frage:
Was ist in diesem unendlichen Gitter von Widerständen der äquivalente Widerstand?
Malabarba
2010-12-20 06:11:13 UTC
view on stackexchange narkive permalink

Ich habe gesucht und konnte es auf der Website nicht finden. Hier ist es (zitiert nach dem Buchstaben):

Auf diesem unendlichen Gitter idealer Ein-Ohm-Widerstände, Was ist der äquivalente Widerstand zwischen den beiden markierten Knoten?

Nerd Sniping

Mit einem Link zur Quelle.

Ich bin mir nicht sicher, ob es eine Antwort auf diese Frage gibt. Aufgrund meines Mangels an Fachwissen über grundlegende Elektronik könnte es jedoch sogar einfach sein.

Ich habe den Titel von XKCD sofort erkannt [Nerd Sniping ist einer meiner Favoriten].
Diskussion über Meta: http://meta.physics.stackexchange.com/q/253/
[Die Frage @ m.SE ...] (http://math.stackexchange.com/questions/12863)
@MarkEichenlaub Zu Ihrem zweiten Kommentar: Ich habe nicht die Zeit oder den Willen, jedes einzelne Ergebnis detailliert darzustellen, aber im Wesentlichen werden drei verschiedene Probleme angesprochen (benachbart, diagonal und "Ritterzug" oder sogar vier, wenn Sie die allgemeine Lösung zählen). . Die Top-Lösung ist über 2800 Wörter lang, geht auf viele mathematische Details ein und löst nur das allgemeine Diagonalproblem. Ich bin der Meinung, dass die Frage immer noch eine präzise, ​​klare und organisierte Antwort benötigt, die leicht zu finden ist.
Es ist ein Duplikat, aber es ist zu spät, um es zu schließen. Ich werde es ein "gutes Duplikat" nennen und in Ruhe gehen.
Verwandte: [Das Problem "Nerd Sniping". Verallgemeinerungen?] (Http://physics.stackexchange.com/questions/10308/)
Zwei antworten:
Sklivvz
2010-12-20 06:38:13 UTC
view on stackexchange narkive permalink

Nerd Sniping!

Die Antwort lautet $ \ frac {4} {\ pi} - \ frac {1} {2} $.

Einfache Erklärung:

sukzessive Annäherung! Ich beginne mit dem einfachsten Fall (siehe Abbildung unten) und füge immer mehr Widerstände hinzu, um ein unendliches Gitter von Widerständen zu approximieren.

Simulation results

Mathematische Ableitung:

$$ R_ {m, m} = \ frac 2 \ pi \ left (1 + \ frac 13 + \ frac 15 + \ frac 17 + \ dots + \ frac 1 {2m-1} \ right) $$

+1, aber es wäre noch besser, die Lösung im Beitrag zu skizzieren, damit die Leute nicht auf einen Link klicken müssen, um zu sehen, wie es gemacht wird.
Das Zeug auf diesem mathematischen Link ist ziemlich kompliziert ... Zu viel für bloße unmenschliche Lebensformen wie mich.
Ja, ich brauchte ein paar Lesungen, um herauszufinden, wie es gemacht wurde. (Das macht es "lustig" :-P)
@David Zaslavsky: In diesem speziellen Fall bin ich mit der numerischen Berechnung wahrscheinlich viel zufriedener als mit der genauen Ableitung.
@Sklivvz: unabhängig davon sollten wir eine Erklärung haben und nicht nur einen Link in der Antwort. (Für Ihre Antwort, wie sie ist, war ich möglicherweise zu schnell, um auf den Upvote-Button zu klicken.)
-1 Das ist definitiv keine einfache Erklärung, ich möchte einfach sehen. Alles, was Sie brauchen, um dies zu lösen, ist das Ohmsche Gesetz zusammen mit dem von Kirchoff.
Erklärung! = Simulation. Ich möchte auch die allgemeine Lösung für zwei beliebige Punkte kennen.
@kalle43 Ich kenne keinen besseren Weg, um das Problem anzugehen als die, die in den Links von @Sklivvz beschrieben sind. Nur weil Sie einfach wollen, heißt das nicht, dass es existiert. Wenn es so einfach ist, warum finden Sie nicht selbst eine einfache Antwort!
@Mark Ich habe bald einen elementaren Beweis, bin mir aber nicht sicher, ob ich ihn hier veröffentlichen soll.
@kalle43: die "Simulation" ist auch eine Erklärung (viel mehr als bloße Mathematik). Wenn Sie die "innere" Schaltung betrachten und immer mehr Schichten hinzufügen, sehen Sie, dass ihr Beitrag immer weniger bedeutend ist. In der Praxis arbeitet die Schaltung also ziemlich lokal und Widerstände, die mehr als 20 Schritte entfernt sind, sind unbedeutend.
@kalle: Kirchhoffs Gesetz ist das, was ich oben in meinem Kommentar erwähnt habe. Sie erhalten eine unendlich dimensionale Matrix und müssen deren Determinante berechnen. Oder Sie können verschiedene Dualitäten verwenden, die das Widerstandsnetzwerk mit Modellen der statistischen Physik verbinden. Trotzdem bezweifle ich sehr, dass eine mögliche Methode in irgendeiner Weise einfach ist. Sie müssen auf jeden Fall irgendwann eine Fourier-Transformation oder nicht triviale Integrale (wie im Link von Sklivvz) durchführen, um die Ergebnisse zu erhalten. Sie sagen also, dass Sie etwas Einfaches erhalten haben, das diese etablierten Methoden übertreffen kann? Ich kann nicht sagen, dass ich nicht an dir zweifle ;-)
@JOHA: Die Ränder hier sind wahrscheinlich sowieso zu klein für Ihren Beweis. ;)
@JOHA: Generalisierung wird [hier] gefragt (http://physics.stackexchange.com/questions/10308/the-nerd-sniping-problem-generalizations)
$ R_ {m, m} $ ist der Widerstand zwischen $ (0,0) $ und $ (m, m) $, d. H. Auf der Diagonale.Die Frage fragt nach $ R_ {2,1} $, daher ist die Aufnahme von $ R_ {m, m} $ in die Antwort unbegründet und sollte wahrscheinlich gelöscht werden.
Und irgendwie bekommen wir dort Pi ohne Kreise.Mathe ist unglaublich.
Ich bevorzuge die Annäherung von 0,773 als Antwort.aber andererseits würde mich dieser Bus wahrscheinlich nie auf traurige Weise treffen.:(
PBS
2018-04-21 20:25:44 UTC
view on stackexchange narkive permalink

Hier ist meine Lieblingsableitung, die ziemlich stark basiert, aber meiner Meinung nach eher einfacher ist als die in den obigen Links angegebenen. Es ist nur eine elementare Integration erforderlich!

Das Setup

Arbeiten Sie der Allgemeinheit halber an einem $ N $ -dimensionalen Raster und beschriften Sie die Rasterpunkte mit $ \ vec {n} $, einem ganzzahligen Vektor.

Angenommen, die Spannung an jedem Punkt ist $ V_ \ vec {n} $. Dann ist der Strom, der von seinen $ 2N $ -Nachbarn in $ \ vec {n} $ fließt,

$$ \ sum_ {i, \ pm} (V _ {\ vec {n} \ pm \ vec {e} _i} - V_ \ vec {n}) $$

($ \ vec {e} _i $ ist der Einheitsvektor entlang der $ i $ -Richtung.)

Bestehen Sie darauf, dass eine externe Quelle einen Ampere in $ \ vec {0} $ und aus $ \ vec {a} $ pumpt. Aktuelle Erhaltung gibt

$$ \ sum_ {i, \ pm} (V _ {\ vec {n} \ pm \ vec {e} _i} - V_ \ vec {n}) = - \ delta_ \ vec {n} + \ delta_ {\ vec {n} - \ vec {a}} $$

($ \ delta_ \ vec {n} $ entspricht $ 1 $, wenn $ \ vec {n} = \ vec {0} $ und andernfalls $ 0 $.)

Dies ist die Gleichung, die wir lösen wollen. Bei $ V_ \ vec {n} $ ist der effektive Widerstand zwischen $ \ vec {0} $ und $ \ vec {a} $ einfach

$$ R_ \ vec {a} = V_ \ vec {0} - V_ \ vec {a} $$

Leider gibt es unendlich viele Lösungen für $ V_ \ vec {n} $, und ihre Ergebnisse für $ R_ \ vec {a} $ stimmen nicht überein! Dies liegt daran, dass die Frage keine Randbedingungen im Unendlichen spezifiziert. Je nachdem, wie wir sie auswählen, können wir einen beliebigen Wert von $ R_ \ vec {a} $ erhalten, den wir mögen! Es wird sich herausstellen, dass es eine einzigartige vernünftige Auswahl gibt, aber lassen Sie uns dieses Problem zunächst vollständig vergessen und jede Lösung finden.

Lösung durch Fourier-Transformation

Die Strategie besteht darin, eine grüne Funktion $ G_ \ vec {n} $ zu finden, die

erfüllt

$$ \ sum_ {i, \ pm} (G _ {\ vec {n} \ pm \ vec {e} _i} - G_ \ vec {n}) = \ delta_ \ vec {n} $$

Eine Lösung für die ursprüngliche Gleichung wäre dann

$$ V_n = -G_ \ vec {n} + G _ {\ vec {n} - \ vec {a}} $$

Um $ G_ \ vec {n} $ zu finden, nehmen Sie an, dass es als

dargestellt werden kann

$$ G_ \ vec {n} = \ int_0 ^ {2 \ pi} \ frac {d ^ N \ vec {k}} {(2 \ pi) ^ N} (e ^ {i \ vec {k } \ cdot \ vec {n}} - 1) g (\ vec {k}) $$

Dann notiere das

\ begin {align} \ sum_ {i, \ pm} (G _ {\ vec {n} \ pm \ vec {e} _i} - G_ \ vec {n}) & = \ int_0 ^ {2 \ pi} \ frac {d ^ N \ vec {k}} {(2 \ pi) ^ N} e ^ {i \ vec {k} \ cdot \ vec {n}} \ left (\ sum_ {i, \ pm} e ^ {\ pm i k_i} - 2N \ right) g (\ vec {k}) \\ \ delta_ \ vec {n} & = \ int_0 ^ {2 \ pi} \ frac {d ^ N \ vec {k}} {(2 \ pi) ^ N} e ^ {i \ vec {k} \ cdot \ vec {n}} \ end {align}

Wir sehen, dass die Gleichung für $ G_ \ vec {n} $ durch Auswahl von

gelöst werden kann

$$ g (\ vec {k}) = \ frac {1} {\ sum_ {i, \ pm} e ^ {\ pm k_i} - 2N} $$

was zu

führt

$$ G_ \ vec {n} = \ frac {1} {2} \ int_0 ^ {2 \ pi} \ frac {d ^ N \ vec {k}} {(2 \ pi) ^ N} \ frac {\ cos (\ vec {k} \ cdot \ vec {n}) - 1} {\ sum \ cos (k_i) - N} $$

Übrigens scheint das lustige $ -1 $ im Zähler nicht viel anderes zu tun, als $ G_ \ vec {n} $ um eine $ \ vec {n} $ - unabhängige Konstante zu verschieben, also Sie könnte sich fragen, was es dort tut. Aber ohne es wäre das Integral unendlich, zumindest für $ N \ leq 2 $.

Die endgültige Antwort lautet also

$$ R_ \ vec {a} = V_ \ vec {0} - V_ \ vec {a} = 2 (G_ \ vec {a} - G_ \ vec {0}) = \ int_0 ^ {2 \ pi } \ frac {d ^ N \ vec {k}} {(2 \ pi) ^ N} \ frac {1 - \ cos (\ vec {k} \ cdot \ vec {a})} {N - \ sum \ cos (k_i)} $$

Warum ist das die richtige Antwort?

(Ab diesem Zeitpunkt ist $ N = 2 $)

Ich habe vorhin gesagt, dass es unendlich viele Lösungen für $ V_ \ vec {n} $ gibt. Das obige ist jedoch etwas Besonderes, da sich die Spannungen und Ströme in großen Entfernungen $ r $ vom Ursprung wie

verhalten

$$ V = \ mathcal {O} (1 / r) \ qquad I = \ mathcal {O} (1 / r ^ 2) $$

Ein Standardsatz (Eindeutigkeit von Lösungen zur Laplace-Gleichung) besagt, dass es nur eine Lösung geben kann, die diese Bedingung erfüllt. Unsere Lösung ist also die einzigartige mit dem geringstmöglichen Stromfluss im Unendlichen und mit $ V_ \ infty = 0 $ . Und selbst wenn die Frage nicht danach gefragt hat, ist es offensichtlich die einzig vernünftige Sache, die gestellt werden muss.

Oder doch? Vielleicht möchten Sie das Problem lieber definieren, indem Sie an einem endlichen Raster arbeiten, dort die eindeutige Lösung für $ V_ \ vec {n} $ finden und dann versuchen, eine Grenze zu setzen, wenn die Rastergröße unendlich wird. Man kann jedoch argumentieren, dass das aus einem Raster der Größe $ L $ erhaltene $ V_ \ vec {n} $ mit einem Fehler in der Reihenfolge $ 1 / L $ zu unserem $ V_ \ vec {n} $ konvergieren sollte. Das Endergebnis ist also dasselbe.

Integrale ausführen

Betrachten Sie zuerst den diagonalen Fall

\ begin {align} R_ {n, n} & = \ frac {1} {(2 \ pi) ^ 2} \ int_A dx \, dy \, \ frac {1 - \ cos (n (x + y))} {2 - \ cos (x) - \ cos (y)} \\ & = \ frac {1} {2 (2 \ pi) ^ 2} \ int_A dx \, dy \, \ frac {1 - \ cos (n (x + y))} {1 - \ cos (\ frac { x + y} {2}) \ cos (\ frac {xy} {2})} \ end {align}

wobei $ A $ das Quadrat $ 0 \ leq x, y \ leq 2 \ pi $ ist.

Da der Integrand periodisch ist, kann die Domäne wie folgt von $ A $ in $ A '$ geändert werden:

Rectangles A and A'

Ändern Sie dann die Variablen in

$$ a = \ frac {x + y} {2} \ qquad b = \ frac {x-y} {2} \ qquad dx \, dy = 2 \, da \, db $$

das Integral wird

$$ R_ {n, n} = \ frac {1} {(2 \ pi) ^ 2} \ int_0 ^ \ pi da \ int _ {- \ pi} ^ \ pi db \, \ frac {1 - \ cos (2na)} {1 - \ cos (a) \ cos (b)} $$

Das $ b $ -Integral kann mit der Halbbräunungssubstitution

durchgeführt werden

$$ t = \ tan (b / 2) \ qquad \ cos (b) = \ frac {1-t ^ 2} {1 + t ^ 2} \ qquad db = \ frac {2} {1+ t ^ 2} dt $$

geben

$$ R_ {n, n} = \ frac {1} {2 \ pi} \ int_0 ^ \ pi da \, \ frac {1 - \ cos (2na)} {\ sin (a)} $$

Die Triggeridentität

$$ 1 - \ cos (2na) = 2 \ sin (a) \ big (\ sin (a) + \ sin (3a) + \ dots + \ sin ((2n-1) a) \ big) $$

reduziert das verbleibende $ a $ -Integral auf

\ begin {align} R_ {n, n} & = \ frac {2} {\ pi} \ left (1 + \ frac {1} {3} + \ dots + \ frac {1} {2n-1} \ right) \ end {align}

Induktion

Während eine Integration erforderlich war, um die Diagonalwerte von $ R_ {m, n} $ zu erhalten, kann der Rest ohne diese ermittelt werden. Der Trick besteht darin, Rotations- / Reflexionssymmetrie zu verwenden,

$$ R_ {n, m} = R _ {\ pm n, \ pm m} = R _ {\ pm m, \ pm n} $$

zusammen mit der folgenden Wiederholungsrelation

$$ R_ {n + 1, m} + R_ {n-1, m} + R_ {n, m + 1} + R_ {n, m-1} - 4 R_ {n, m} = 2 \ delta _ {(n, m)} $$

, das mit $ R_ \ vec {n} = 2 G_ \ vec {n} $ und der Definitionsgleichung für $ G_ \ vec {n} $ abgeleitet werden kann.

Beginnen Sie mit der trivialen Aussage, dass

$$ R_ {0,0} = 0 $$

Das Anwenden der Wiederholungsrelation bei $ (0,0) $ und das Verwenden der Symmetrie ergibt

$$ R_ {1,0} = R_ {0,1} = 1/2 $$

Die nächste Zeile wird wie folgt ausgeführt

Fill in R11, then R02 and R20

Und der danach ...

Fill in R12 and R21, then R03 and R30

Das wiederholte Abwechseln der beiden obigen Schritte ergibt einen Algorithmus zum Bestimmen jedes $ R_ {m, n} $. Es ist klar, dass alle die Form

haben

$$ a + b / \ pi $$

wobei $ a $ und $ b $ rationale Zahlen sind. Jetzt kann dieser Algorithmus leicht von Hand ausgeführt werden, aber man kann ihn auch in Python codieren:

  importiere numpy als np
Bruchteile importieren als fr

N = 4
arr = np.empty ((N * 2 + 1, N * 2 + 1, 2), dtype = 'Objekt')

def plus (i, j):
    arr [i + 1, j] = 4 * arr [i, j] - arr [i - 1, j] - arr [i, j + 1] - arr [i, abs (j - 1)]

def sogar (i):
    arr [i, i] = arr [i - 1, i - 1] + [0, fr.Fraction (2, 2 * i - 1)]
    für k im Bereich (1, i + 1): plus (i + k - 1, i - k)

def ungerade (i):
    arr [i + 1, i] = 2 * arr [i, i] - arr [i, i - 1]
    für k im Bereich (1, i + 1): plus (i + k, i - k)

arr [0, 0] = 0
arr [1, 0] = [fr.Fraction (1, 2), 0]

für i im Bereich (1, N):
    selbst ich)
    ungerade (i)

gerade (N)

für i im Bereich (0, N + 1):
    für j im Bereich (0, N + 1):
        a, b = arr [max (i, j), min (i, j)]
        print ('(', a, ') + (', b, ') / π', sep = '', end = '\ t')
    drucken()
 

Dies erzeugt die Ausgabe

$$ \ Groß \ begin {array} {| c: c: c: c: c} 40 - \ frac {368} {3 \ pi} & \ frac {80} {\ pi} - \ frac {49} {2} & 6 - \ frac {236} {15 \ pi} & \ frac {24} {5 \ pi} - \ frac {1} {2} & \ frac {352} {105 \ pi} \\ \ hdashline \ frac {17} {2} - \ frac {24} {\ pi} & \ frac {46} {3 \ pi} - 4 & \ frac {1} {2} + \ frac {4} {3 \ pi } & \ frac {46} {15 \ pi} & \ frac {24} {5 \ pi} - \ frac {1} {2} \\ \ hdashline 2 - \ frac {4} {\ pi} & \ frac {4} {\ pi} - \ frac {1} {2} & \ frac {8} {3 \ pi} & \ frac {1} {2}+ \ frac {4} {3 \ pi} & 6 - \ frac {236} {15 \ pi} \\ \ hdashline \ frac {1} {2} & \ frac {2} {\ pi} & \ frac {4} {\ pi} - \ frac {1} {2} & \ frac {46} {3 \ pi} - 4& \ frac {80} {\ pi} - \ frac {49} {2} \\ \ hdashline 0 & \ frac {1} {2} & 2 - \ frac {4} {\ pi} & \ frac {17} {2} - \ frac {24} {\ pi} & 40 - \ frac {368} {3 \ pi} \\ \ hline \ end {array} $$

von dem wir die endgültige Antwort ablesen können,

$$ R_ {2,1} = \ frac {4} {\ pi} - \ frac {1} {2} $$

Es ist sehr interessant, dass der gleiche Formalismus verwendet werden kann, um die Zweipunktfunktion eines masselosen Skalarfelds auf einem Gitter zu lösen
Du hast die Einheiten vergessen


Diese Fragen und Antworten wurden automatisch aus der englischen Sprache übersetzt.Der ursprüngliche Inhalt ist auf stackexchange verfügbar. Wir danken ihm für die cc by-sa 2.0-Lizenz, unter der er vertrieben wird.
Loading...